You are on page 1of 66

1A

2010-AL PHY

HONG KONG EXAMINATIONS AND ASSESSMENT AUTHORITY HONG KONG ADVANCED LEVEL EXAMINATION 2010

PHYSICS A·LEVEL PAPER 1 Question·Answer Book 1 A

8.30 am - 11.30 am (3 hours)

This paper must be answered in English

INSTRUCTIONS

(1) After the announcement of the start of the examination, you should first write your Candidate Number in the space provided on Page 1 and stick barcode labels in the spaces provided on Pages 1, 3 and 5.

(2) This paper consists of TWO sections, A and B, and

each section carries 54 marks. Answer ALL the

questions in each section.

(3) Questions for sections A and B are printed in two separate Question-Answer Books, 1A and 1 B respectively.

(4) Write your answers in the spaces provided in the Question-Answer Books. Do not write in the margins. Answers written in the margins will not be marked.

(5)

Graph paper and supplementary answer sheets will be supplied on request. Write your Candidate Number, mark the question number box and stick a barcode label on each sheet, and fasten them with a string INSIDE this book.

(6)

Question-Answer Books 1 A and 1 B must be handed in separately at the end of the examination.

(7) No extra time will be given to candidates for sticking the barcode labels or filling in the question number boxes after the 'Time is up' announcement.

2010-AL-PHY IA-I

78

Please stick the barcode label here.

Icandidate Numberl

I I I I I I

Marker's Use Examiner's
Only Use Only
Marker No. Examiner No.

Question No. Marks Marks
,
1 , ,
,
, :
,
2
3
·
·
·
·
4 ,
,
Section A ,
Total , Checker's Use Only

Checker No.

Total

-ci A hemisphere with centre 0 is fixed on the ground as shown in Figure 1.1. It has a smooth surface of tl)

~

radius r. A bead of mass m is placed at the top of the hemisphere. When slightly disturbed, the bead starts to a

slide from rest on the surface fromXto Ywhile its angular displacement is e. (Neglect air resistance.)

SECTION A

Answer ALL questions.

ii Write your answers in the spaces provided in this Question-Answer Book. You should show all the main steps of your calculations.

iii Assume: speed of light in air = 3 x 108 m S-1

acceleration due to gravity = 10 m S-2

Question No.

1

2

3

4

Marks

14

13

13

14

1.

Figure 1.1

(a)

By considering the energy of the system, express the speed of the bead v at Y in terms of e and r.

(2 marks)

-

-

.~

en

Q

.~ .-----------------------------------------------------------------------------------------------------------------------------------------------------------------

S

.s .-----------------------------------------------------------------------------------------------------------------------------------------------------------------

.S

._---------------------------------------------------------------------------------------------------------------------------------------------------------.---.--

.~

~ .. __ .. - .. _. __ . . . ._ _ . __ . _ __ . . .

Draw and label the force(s) acting on the bead at Y in Figure 1.1.

(b)

(2 marks)

(c)

By considering the circular motion of the bead on the hemisphere and using the result in (a), show that the reaction force acting on the bead at Y is given by (Acose - B)mg and fmd the constants A and B. Hence use the result to explain the variation of the reaction force with e. (4 marks)

Answers written in the margins will not be marked.

Page Total

2010-AL-PHY lA-2

79

.....

o Q

.~

Please stick the barcode label here.

(d) The bead eventually leaves the hemisphere and hits the ground.

(i) Find the value of () at which the bead just leaves the hemisphere. Describe its subsequent

trajectory before hitting the ground. (4 marks)

-d

(1)

] ------------------------_._--------------------------------------------------------------------------------------------------------------------------------------

8

(1)

.0 .-----------------------------------------------------------------------------------------------------------------------------------------------------------------

......

o ~

- ---------------_------._----------------------------------------------_._----------------------------------------------------------------------------------------

-

.~

ell

~ ._----------------------------------------------------------------------------------------------------------------------------------------------------------------

.~

8 -----------------------------------------------------------------------------------------------------------------------------------------------------------------

(1)

;S

.S -----------------------------------------------------------------------------------------------------------------------------------------------------------------

.~ ._-----------------------------------------------------------------------------------------_----------------------------------------------------------------------

....

~

(ii)

Calculate the speed of the bead just before it hits the ground if the radius r of the hemisphere isO.lm. (2 marks)

Answers written in the margins will not be marked.

Page Total

20IO-AL-PHY lA-3

80

.~

til

.S

~

8

(1)

;S

.S

~ (1)

.E

....

~

ell .... (1)

~

ell

~

<

metre rule

2.

Figure 2.1

An LED (Light Emitting Diode) emitting monochromatic light of wavelength A is viewed through a diffraction grating of 160 lines per min as shown in Figure 2.1. With the aid of a pencil and two mutually perpendicular metre rules, several positions of images corresponding to the maxima are located in the way shown. Figure 2.2 shows the observation through the diffraction grating.

-d
Q) LED
..::.:
a 0
a I I
Q)
.D Oem
.....
0
r::
-
.~
en
.S
01) (a) (i)
....
o:s
a
Q)
..r::
.....
. S
r::
Q)
.t:!
....
~
en
....
Q)
~
en
r::
<t: images of LED
0 0 0 0
I I I I I I I I I I I I I I I
10 20 30 40 50
Figure 2.2 .~

Order of maxima n 1 2 3 4
Angular position of images () / 0 en

.S Find the angular positions of images () according to the above observation. Complete the ~

table by choosing a suitable physical quantity so as to obtain a straight line graph. Plot the a graph on Page 5 and fmd A. (6 marks) .g

.S

r::

~

.;::

~

~ - - - - - - - - - - - - - - - - - - - - - - - - - - - - - - - - - - - - - - - - - - - - - - - - - - - - - - - - - - - - - - - - - - - - - - - - - - - - - - - - - - - - - - - - - - - - - - - - - - - - - - - - - - - - - - - - - - - - - - - - - - - - - - - - - - - - - - - - - - - - - - - - 00 _

Answers written in the margins will not be marked.

2010-AL-PHY lA--4

81

Page Total

Please stick the barcode label here.

-d
11)
~
s
11)
.D
.....
0
:::
-
-
.~
til
:::
'on
.....
ro
S
11)
..c:
.....
. S
:::
11)
.i.:l
.....
~
til (ii)
.....
11)
~
til
:::
-< .~

til

.S

~

S

11)

-;S

.S

.~

.....

~ If a laser is used instead of an LED in this experiment, sketch the set-up to show the ~

modification required. State ONE safety precaution, and ONE precaution for getting accurate ~ results. (3 marks) ~

[Turn over to page 6 for question 2(b)]

Answers written in the margins will not be marked.

2010-AL-PHY lA-5

82

Page Total

(ii)

<IJ

.;3 The energy of each photon emitted by the LED is 1.85 eV. Using the wavelength A found in .5

(a)(i), estimate the value of the Planck constant h. (2 marks)

(b)

An LED is made up of two different semiconductor layers, namely p-type and n-type. Assume that one photon is emitted when an electron passes through the LED. Figure 2.3 shows the circuit for lighting up the LED. Given: charge of an electron e = 1.60 x 10-19 C

2.

~--------~11--------~

Figure 2.3

(i) When the current in the LED is 8.0 rnA, estimate the number of photons emitted by the LED

per second. (2 marks)

-

-- ._--------------------------------------_---------_--_------------------------------------.-----------------.--------------------------------.--------------------

.~

'"

.5

~

S

-------------------------------------------------------------------------------------------------------------------------------------------.--------------------- .. S

~

s

'"

~

~ ... _-------------------------------------------------------_._----------------_._--.------------------------------------------------------------------------------

~

Answers written in the margins will not be marked.

Page Total

2010-AL-PHY lA-6

83

.~

This is a blank page.

2010-AL-PHY lA-7

84

(2 marks)

3. A student uses a Kundt' s tube to measure the speed of sound in air. The long glass tube has one end closed and the other end fitted to a loudspeaker. Lycopodium powder is spread evenly inside the tube initially. The loudspeaker is driven by a signal generator to produce sound. The frequency of the signal generator is gradually increased until heaps of powder are formed (Figure 3.1) at the frequency 2000 Hz.

Kundt's tube

Figure 3.1

To signal genecatoc~

loudspeaker ;t -l.- __ ;-'\. ..... ':_~-=--=--:_":_-=--=-:_""""-_.... -_ -_ -_:'_~_-~_ ..... ::,/./-:-------'

heaps of lycopodium powder Explain why heaps of powder are formed.

(2 marks)

(a)

(i)

(ii) Figure 3.2(a) shows part of the variation of maximum displacement of air molecules from their equilibrium positions with distance along the tube. Displacement to the right is taken to be positive. Sketch on Figure 3 .2(b) the corresponding variation of air pressure with distance at that instant. (2 marks)

maximum displacement

Figure 3.2(a)

right 0 V\ /\

M

no:;,""ure I !

P,."urell· I i

Of---~--+--~--+--+

left

Figure 3 .2(b)

distance along the tube

(b)

The separation between the first and the fifth heaps is found to be 0.35 ± 0.02 m.

(i) Estimate the speed of sound in air.

Answers written in the margins will not be marked.

Page Total

2010-AL-PHY lA-8

85

-

.~

(ii) Suggest a reason why the error in the above measurement of heap separation is quite large.

The percentage error in reading the frequency from the dial of the signal generator is 2%. What is the maximum possible percentage error in the speed of sound estimated in (b )(i) ?

(3 marks)

(iii) The student increases the power of the signal generator to produce a sound of greater loudness without altering the frequency. Explain whether the positions of heaps in the tube would change. (2 marks)

(c) The Kundt's tube is now removed. The loudspeaker is driven to produce a sound of intensity level 60 dB at a certain point P. If the intensity level of background noise is also 60 dB, estimate the sound intensity level reading of a dB-meter placed at P. (2 marks)

Answers written in the margins will not be marked.

Page Total

2010-AL-PHY lA-9

86

(ii) The energy released in the decay becomes the kinetic energy of the decay products. Explain quantitatively why the a-particle takes most of the energy and find its speed v. Assume that the parent nucleus is at rest initially. (4 marks)

4.

(a)

A nucleus of radon (2~~ Rn) decays to an isotope of polonium (Po) by emitting an a-particle.

Given: mass ofa radon nucleus = 222.0176 u mass of a polonium nucleus = 218.0090 u mass of an a-particle = 4.0026 u

1 u = 1.66 x l 0-27 kg, which is equivalent to 931 MeV charge of an electron e = 1.60 x 10-19 C

(i) Write an equation for the decay and find the energy released, in MeV, in the decay. (3 marks)

-

-

.~

Vl

.S

~

E

~

.S

.~

~

t

~

Vl ~ ..-:t:

Answers written in the margins will not be marked.

Page Total

201O-AL-PHY lA-lO

87

-

.~

-

.~

(b)

-

.~

(i)

a-particles with the speed v found in (a)(ii) are directed into an evacuated region with a uniform magnetic field B = 0.5 T normal to the initial velocity of the a-particles as shown in Figure 4.1. Find the radius of the path described by the a-particles in the field region.

Given: the charge to mass ratio of an a-particle is 4.82 x 107 C kg " (2 marks)

-d

Q)

~

S

Q) .D

.....

o ~

(ii) Explain whether the a-particles would emerge with a greater speed from the field. (2 marks)

(iii)

In 1909, Rutherford collected some a-particles which then changed into atoms of a gas by receiving electrons from the surroundings. The gas was trapped inside an evacuated tube with electrodes A and B as shown in Figure 4.2. Explain how Rutherford could prove that a-particles are actually helium nuclei by using this set-up. (3 marks)

A

evacuated tube containing the gas

Figure 4.2

B

END OF SECTION A

Sources of materials used in this paper will be acknowledged in the Examination Report and Question Papers published by the Hong Kong Examinations and Assessment Authority at a later stage.

Answers written in the margins will not be marked.

Page Total

201O-AL-PHY lA-II

88

18

2010-AL PHY

HONG KONG EXAMINATIONS AND ASSESSMENT AUTHORITY

HONG KONG ADVANCED LEVEL EXAMINATION 2010

PHYSICS A·LEVEL PAPER 1 Question·Answer Book 1 B

8.30 am - 11.30 am (3 hours)

This paper must be answered in English

INSTRUCTIONS

(1) After the announcement of the start of the examination, you should first write your Candidate Number in the space provided on Page 1 and stick barcode labels in the spaces provided on Pages 1, 3 and 5.

(2) No extra time will be given to candidates for sticking the barcode labels or filling in the question number boxes after the 'Time is up' announcement.

2010-AL-PHY IB-l

89

Please stick the barcode label here.

Icandidate Number/

/ / / / / /

Marker's Use Examiner's
Only Use Only
Marker No. Examiner No. Question No. Marks Marks
, ,
5 ,
,
,
, ,
,
6 ,
,
,
7 , ,
, ,
8 , ,
, ,
, ,
,
Section B ,
, ,
Total ,
,
, Checker's Use Only

Checker No.

Total

SECTION B

Answer ALL questions.

ii Write your answers in the spaces provided in this Question-Answer Book. You should show all the main steps of your calculations.

iii Assume: speed of light in air = 3 x 108 m S-1

acceleration due to gravity = 10 m S-2

Question No.

6

5

7

8

Marks

10

2010-AL-PHY 18-2

13

12

19

5. Figure 5.1 shows the variation of the gravitational potential V due to an asteroid with distance r from its centre, which is plotted from the asteroid's surface outwards. Assume that the mass distribution of the asteroid is spherically symmetric.

Figure 5.1

0
-2000
-4000
-6000
eo
"" -8000
~
~
-10000
-12000
-14000 Gravitational potential of the asteroid

lxl05 2xl05

-~

r (m)

/

/

v

-16000 -------- ------.-_._-- --.---.---- ----. _._L --_.-.--.-._-

asteroid's surface

(a)

Find the weight ofa rock of mass 60 kg on the asteroid's surface.

(3 marks)

(b) An international space agency sends a probe to the asteroid. The probe then moves into a circular orbit around the asteroid at a distance r = 2 x 105 m in order to investigate the asteroid. Use Figure 5.1 to find the orbital speed and period of the probe. ( 4 marks)

._----------------------------------------------------------------------------------------------------------------------------------------------.-----------------

Answers written in the margins will not be marked.

90

Page Total

.~

Please stick the barcode label here.

~ ~

<L> •.••.•••••••••••••.•••••••••••...••••.•..••••••••••••..•••••••••••••••••••••••••••••••••••••••••••••••.••••••••••••••••••.•••••••••••••••••••••••••••••••.••••••• <L>

~ ~

s s

(c)

(i)

Estimate from Figure 5.1 the escape velocity from the asteroid's surface.

(2 marks)

<L> .0

'0

s::

-

-

.r;;

~ ~

s:: s::

.~ ~

s S

<L>

-5 -.-.------.-------.-.--------.-- .. - -.-------------------- .. ---.------------.-.----.-- -.-.-----.-

.s

.5

.5

.~ ._---_._--------_ _ _. __ _ _ _ - _._--_ ~

~ ~

CIl .------------------.------------------------------------------------------------------------------------------.-------------------------------------------------- •.

.... <L>

~

~

(ii)

t

~

'" s::

The daytime surface temperature of the asteroid is found to be 200 K. Estimate the typical < r.m.s. speed of gas molecules, say, nitrogen on the asteroid's surface. Hence explain whether

the asteroid can retain any atmosphere. (4 marks) Given: Boltzmann constant k = 1.38 X 10-23 J K-1

Mass of a nitrogen molecule = 4.65 x 10-26 kg

Answers written in the margins will not be marked.

91

2010-AL-PHY IB-3

Page Total

-d .-----------------------------------------------------------------------------------------------------------------------------------------------------------------

Q)

]

E

Q) ~

(5 !:::

6. This question describes how a typical radio picks up radio signals. Radio wave is a kind of electromagnetic wave which consists of varying electric and magnetic fields. The schematic diagram in Figure 6.1 shows a

radio wave passing a metal rod with its electric field E parallel to the rod.

E

metal rod

x

Figure 6.1

y

(a) By considering the motion of free electrons in the metal rod, explain why an alternating voltage is

induced across ends XY. (2 marks)

(b)

The alternating voltage V picked up by the rod is fed to the LRC circuit shown in Figure 6.2. The resistance of R is 10 Q and the inductance of L is 3.5 X 10-4 H. The capacitance of the variable capacitor C is set at 5.0 xlO-11 F.

-

-

.~

'"

.S

~

E

Q)

;S

.S

.~

....

~

3.5xlO-4H ~

~

Find the resonant frequency fa and the impedance of the LRC circuit at resonance. (3 marks) ~

R

.~ sg

.~

E

Q)

;S

.S Figure 6.2

.~

~

C

10 Q

(i)

Answers written in the margins will not be marked.

2010-AL-PHY IB-4

92

Page Total

Please stick the barcode label here.

~ ~

o 0

I:::: .•••••• • ------- . I::::

(ii) If the amplitude of V is 2.5 m V and its frequency is exactly equal to the resonant frequency

of the LRC circuit, find the corresponding amplitude of the voltage across C. (3 marks)

If the maximum capacitance of Cis 1.0 X 10-10 F, what is its minimum value?

(2 marks)

-

-

.~

<Jl

.S

~

S .. ----------------------------------------------------------------------------------------------------------------------------------------------------------------

(iii)

-

Hence, or otherwise, explain why the LRC network can be used to tune a radio to receive ~

radio wave of a particular frequency. (2 marks) <Jl .S

~

S

.S .-----------------------------------------_---------------------------------------------------------------------------------------.-------------------------------

I::::

.~ ._----------------------------------------------------------------------------------------------------------------------------------------------------------------

....

~

<Jl .... <l)

~

~ ._----------------------------------------------------------------------------------------------------------------------------------------------------------------

(c) The resonant frequency of the LRC circuit in (b) can be adjusted by changing the capacitance of the variable capacitor C, which has a fixed metal plate P and a movable metal plate Q_ Both plates are semi-circular in shape and plate Q can be rotated between two extreme positions as indicated in

Figure 6.3_ Q

Figure 6.3

P

P

Answers written in the margins will not be marked.

201O-AL-PHY IB-5

93

Page Total

.S In Fig. 7.1 (b), draw and label the force( s) acting on the piston and find the pressure of the

helium gas. (4 marks) .~

~

~

Q)

~

::: <C

-

.~

(i)

7. A circular piston of mass 0.5 kg suspended by a light spring of force constant 50 N m-I is free to move inside a vertical smooth cylinder with a cross-sectional area of 0.01 m2• When all the air is evacuated from the cylinder as shown in Fig. 7.I(a), the height of the piston above the bottom of the cylinder is 0.20 m.

Fig. 7.1 (a)

Fig. 7.1 (b)

Fig. 7.1 (c)

vacuum

0.5 kg piston

cylinder

L-.-_----l ! 0.20 m helium gas at 300 K

(a)

What is the extension of the spring in Fig. 7.I(a)?

._;

Q)

] .------------------------------------------------------------------------.----------------------------------------------------------------------------------------

S

] ._------------------------_._-------------------------------------------_._---------------------------------------------------------------_._---------------------

-

o

:::

<Il

.5 .-~.--------------------------------------------------------------------------------------------------------------------------------------------------------------

~

S

(b)

When a small amount of helium gas at temperature 300 K is slowly introduced to the space below the piston, the height becomes 0.24 m as shown in Fig. 7.I(b).

t

~ .-------------------------------------------------------------.---------------------------------------------------------------------------------------------------

<Il

:::

<C

Answers written in the margins will not be marked.

Page Total

2010-AL-PHY IB--{)

94

(2 marks)

-

.~

<Il

.S

~

S

Q)

-B

(ii) The helium gas is gradually heated to a temperature T such that the piston rises very slowly to a height of 0.30 m as shown in Fig. 7.I(c). The pressure of the gas becomes 500 Pa. Find the temperature T. (2 marks)

-ci

CI)

(iii) On the given p- V diagram, sketch a graph to show the process undergone by the helium gas ~

in (b)(ii). Mark suitable values on both axes. (3 marks) S

pressure p/Pa

~~~I~-T--I-- r-- tr - -r--j

=t- =-L- ~~ .l.L

I I I i I

"t ----t -t--------r--t---- -I

o

volume V1l0-3 nr'

(c) Consider the process undergone by the helium gas in (b)(ii),

(i) calculate the work done by the gas and the gain in gravitational potential energy of the piston

in the process. Account for the difference in their values, if any. (4 marks)

._------------------------------------------------------------------------------------------------------------------------------------------------_---------------

[Turn over to page 8 for question 7(c)]

Answers written in the margins will not be marked.

Page Total

2010-AL-PHY IB-7

95

-

'r;

7.

(c)

(ii)

find the change in internal energy of the gas in the process. Helium gas is monatomic and is assumed to be ideal. (2 marks)

-d (iii) determine the heat absorbed by the gas in the process. (2 marks)

~

S .

o

i::: .•.•••••••..••••••••••••••••••••••••••••••••••••••••••••••••••••••••••••••••...•••••••••••••.•.••••••••••••••••••••.•...•..••••••••••••••••••.•••••••••••••..•••••

til

.5

!a .

S

(1)

;S .

. S

i:::

.~ .-----------------------------------------------------------------------------------------------------------------------------------------------------------------

~

e

(1)

~

c.n . _

~

Answers written in the margins will not be marked.

Page Total

2010-AL-PHY IB-8

96

This is a blank page.

2010-AL-PHY IB-9

97

8.

(a)

Figure 8.1 shows a rectangular semiconductor chip of length l, width h and thickness t which contains n free electrons per unit volume. Given: the charge of an electron is e.

Figure 8.1

uniform magnetic field B normal to the chip

(i) A uniform magnetic field B is applied normally to the chip carrying current 1. In Figure 8.1

mark the polarity of the Hall voltage induced across the chip. (1 mark)

(ii)

Derive, from first principles, the Hall voltage V in terms of B, I, n, e and the dimensions of the chip. If B is the horizontal component of the Earth's magnetic field at a certain place, which is 30 J.lT acting into the paper, find the Hall voltage induced across the chip when

l = 10.0 mm, h = 3.0 mm, t = 0.15 mm, n = 8.0 x 1019 m-3, 1= 0.05 A and e = 1.60 x 10-19 c. -d

(4 marks) ~ S

-

-

.~ .----------------------------------------------------.---------------------------------------------------------------------------------.--------------------------

en

.s

~

a

.-.----------------------------------------------------------------------------------------------------------.----------------------------------------------------

II)

-fl

.s ------------.-.---------------.-.----.--------------------- -.-.-------------------------------------------.--------.- -------- ---

.1 . . ._._. .... .. . . . . .. . . ._._. . __ . ._. _

:::

t

::: .. -- .. ---.----.-.------.---- ... ------.------.-------------.------------ .... ------.--------------.--.---------.---.-------------------.--------.--.----------.-.---

en

~

Answers written in the margins will not be marked.

Page Total

2010-AL-PHY lB-IO

98

II) .0

-

o

r:::

-

-

.~

en

.s

~

s

II)

-fl .s

·f

t ~

r:::

<

wire Y

(b) Figure 8.2 shows an infinitely long vertical straight wire Y carrying a current of 10 A. P, Q are two points on either side of the wire and both are at distance 0.1 m from the wire. The horizontal component of the Earth's magnetic field around the wire is still 30 IlT acting into the paper. Given: permeability of free space u, is 41t X 10-7 H m-l

Figure 8.2

Q x 10 AA X P

L~/~ ~'~/~ 71

I' / <,

0.1 m 0.1 m

(i)

Calculate the magnetic flux density produced by the current at point P.

(2 marks)

~ ~

<l) .• ._ _._ __ __ .. _ _ .. ._ _ __ .. _ _ _ __ _ .. __ <l)

~ ~

S S

]

....

o

= _ _ .. _ _--._-.- .. _------ .. --_ __ . __ ._ _ .

-

-

.~

-

.~

CI:l .-----------------------------------------------------------------------------------------------------------------------------------------------------------------

.S

bIl

a s

(ii)

If the small semiconductor chip in (a) carrying 0.05 A of current is placed in turn with its centre at P and Q such that the magnetic field due to Y is normal to it, explain whether the Hall voltages induced across it are the same in magnitude. (3 marks)

<l)

-5 - --- -- -.- .. - -.-.---- .. - ---.- .. ---.-.- --.-. _

.S

til

~ ~

til

;e .. - - .. -- -- - - -- - - .. - .. -- -- .. --- ------- - .. -- .. -

END OF PAPER

Sources of materials used in this paper will be acknowledged in the Examination Report and Question Papers published by the Hong Kong Examinations and Assessment Authority at a later stage.

Answers written in the margins will not be marked.

99

201O-AL-PHY lB-ll

Page Total

2010-AL PHY PAPER 2

HONG KONG EXAMINATIONS AND ASSESSMENT AUTHORITY HONG KONG ADVANCED LEVEL EXAMINATION 2010

PHYSICS A·LEVEL PAPER 2

1.30 pm - 4.30 pm (3 hours) a.p. Code 8262 I

INSTRUCTIONS FOR SECTION A

1. Read carefully the instructions on the Answer Sheet. After the announcement of the start of the examination, you should first stick a barcode label and insert the information required in the spaces provided. No extra time will be given for sticking the barcode label after the 'Time is up' announcement.

2. When told to open this book, you should check that all the questions are there. Look for the words 'END OF SECTION A' after the last multiple-choice question.

3. All questions carry equal marks.

4. You are advised to use an HB pencil to mark all your answers on the Answer Sheet, so that wrong marks can be completely erased. You must mark the answers clearly; otherwise you will lose marks if the answers cannot be captured.

5. You should mark only ONE answer for each question. If you mark more than one answer, you will receive NO MARKS for that question.

6. No marks will be deducted for wrong answers.

INSTRUCTIONS FOR SECTION B

7. Look for the words 'END OF PAPER' after the last essay question.

8. Answer any THREE questions from this section. Write your answers in the answer book provided.

Not to be taken away before the end of the examination session

2010-AL-PHY 2-1

100

Section A

There are 45 questions in this section. Each question is followed by four suggested answers.

Where necessary, take the acceleration due to gravity to be 10 m S-2 and take the speed of light in air to be 3 x 108ms-1•

1. An L-shape block XYZ of weight 30 N rests on a rough horizontal surface. The centre of gravity of the block is at G. When a horizontal force P is applied to it at X as shown, the block just overturns when P equals 17 N. If P is applied to the block at X in the opposite direction, what is its minimum value needed to overturn the block?

30N

A. 7N

B. 9N

c. 13 N

D. 17N

2.

® ball

sudden push

I :; R)f--------I()J X

smooth ground

A small ball is initially held above a trolley XY as shown. The trolley is then given a sudden push while the ball is released from rest. The ball leaves a mark on the trolley every time when it hits the trolley. All collisions are perfectly elastic. Which diagram correctly shows the marks on the trolley? (The friction between the ball and the trolley is assumed negligible.)

A.

y













X

B.

y •••



• • •

X

c.

y

•••••





X

D.



•••••

X

2010-AL-PHY 2-2

101

3.

metal frame

sharp point as a pivot

balancing weight

A balancing toy consisting of a metal frame and two heavy balancing weights rests on a platform as shown. The toy can swing back to its equilibrium position when it is tilted slightly. Which of the following can explain this?

(1) The toy's centre of gravity is below the pivot at its equilibrium position. (2) Restoring moment is produced by the balancing weights.

(3) The net force acting on the toy is always zero.

A. B. C. D.

(2) only (3) only

(1) and (2) only (1), (2) and (3)

4.

.... .... -- ......

, , ,

,

,

,

, , ,

p

A small ball is projected from P on the ground. It rebounds at Q on a slope and travels back to P along the same path. Which statements about the ball must be correct?

(I) It hits the slope at Q normally.

(2) It undergoes perfectly elastic collision at Q.

(3) The time taken for the ball to go from P to Q is equal to that for it to return from Q to P.

A. (1) and (2) only

B. (1) and (3) only

C. (2) and (3) only

D. (I), (2) and (3)

5.

P0---+-------~~---

y

Lx

A sphere P moving with a certain speed collides obliquely with another identical sphere Q which is stationary initially. After collision, spheres P and Q move at right angles to each other as shown. Which of the following statements is/are correct? (Neglect friction.)

(I) The x-component of their total linear momentum is conserved. (2) The y-component of their total linear momentum is conserved. (3) Their total kinetic energy is conserved.

A. B. C. D.

(1) only (3) only

(I) and (2) only (1), (2) and (3)

20IO-AL-PHY 2-3

102

I

:0 ground

~~

6.

The figure shows a small object suspended by a light inextensible string from the ceiling. The object is swinging in a vertical plane and every time when it passes the lowest point, a tiny piece of mass is detached from its bottom at that instant. Which of the following diagrams correctly shows the landing positions on the ground of the first four. pieces of mass, namely 1, 2, 3, 4? 0 indicates the position directly under the lowest point of the swing. (Neglect air resistance and friction.)

A. 1,2,3,4 ground
t
0
B. 2,4 1,3 ground
t
0
2 4 3 ground
c. t
0
2 4 3 ground
D. t
0
7.
~2Nm-'
~2222QQQQ~ 0.5 kg A horizontal light spring of force constant 2 N m-' is fixed at one end and a block of mass 0.5 kg is attached to its other end. The block is set into oscillation on a smooth horizontal surface with amplitude

0.1 m. What is the maximum speed of the block? (Neglect air resistance.)

A. 0.14ms-'
B. 0.20 m s-'
c. 0.28 m s-'
D. 0.40 m s-' 8.

, ,

I

I

~---------~~

~----~~ P

,'0· '.

\, I

smooth horizontal surface

------

I I

...... _-------_ ......

Two identical small spheres P and Q are connected by light inextensible threads to a fixed point 0 as shown. The threads OP and PQ are of the same length. P and Q perform uniform horizontal circular motion about 0 with the same period. T" T2 denote the tensions in the threads OP, PQ respectively. Find T, : T2.

A. B. C. D.

3:2 2: 1 3 : 1 4: I

201O-AL-PHY 2-4

103

9. A mass m is suspended by a vertical light spring fixed at its upper end. The force constant of the spring is k. The mass is made to oscillate vertically with amplitude A. Its period of oscillation is T. Which of the following changes can increase the period to 2T?

A. increasing the amplitude to 2A

B. increasing the mass to 4m

C. reducing the force constant to !:.. and increasing the mass to 2m 4

D. increasing the force constant to 2k and reducing the mass to m

2

10. A light spring is hung from the ceiling and a mass is attached to its lower end. The extension of the spring is Xl at static equilibrium. The mass is then supported by a hand such that the spring is just unstretched. The hand is withdrawn suddenly and the mass falls to the lowest position at which the extension of the spring is X2' Find Xl : X2'

A. 2: 3

B. 1 : 3

C 1:2

D. I : I

11.

S

a ----------- .. ----- b

2r r

A comet X moves around the Sun S in an elliptical orbit. Its speed at the closest point b is double that at the farthest point a. The distance of point a from the Sun is 2r while that of point b is r. If the kinetic energy and potential energy of the comet at point a are K and U respectively, deduce the relationship between K and U. (Take gravitational potential energy to be zero at infmity.)

A. U=-K
B. U=-2K
C. U=-3K
D. U=-4K 12. One of Jupiter's moons is called Ganymede. The radius of Ganymede's orbit around Jupiter is about 3 times that of the Moon around the Earth. The mass of Jupiter is 318 times that of the Earth. If the period of the Moon around the Earth is 27.3 days, estimate the period of Ganymede revolving around Jupiter.

A. B. C. D.

2.7 days 8.0 days 91 days 273 days

201O-AL-PHY 2-5

104

13.

y/cm

The figure shows a snapshot of part of a continuous wave of frequency 5 Hz travelling to the right along the x-axis, Particles P and Q are at x = 4 em and x = 8 em respectively. After how long will particle P first have the same displacement and velocity that particle Q has at the instant shown ?

A. 0.08 s

B. 0.12 s

C. 0.14s

D. 0.16 s

14.

Figure (a)

Figure (b)

In a ripple tank, S, and S2 are two coherent sources vibrating with the same frequency. Figure (a) shows the pattern of water waves produced by S, and S2 at time t = O. The solid lines represent the crests of the water waves. Figure (b) shows the displacement-time graph of a particle at P in the pattern. Which of the following displacement-time graphs is possible for a particle at R about mid-way between PQ as shown ?

A.

B.

C.

D.

2010-AL-PHY z-s

105

15.

sound waves

air

water

Sound waves of frequency 1000 Hz travel from air to water as shown. If the incident plane wavefront makes an angle of 130 with the interface, find the angle of refraction and the wavelength of sound in water. (Given: speed of sound in air and that in water are 340 m S-1 and 1500 m S-l respectively)

angle of refraction wavelength in water
A. 2.90 7.7cm
B. 2.90 1.5 m
C. 830 7.7cm
D. 830 1.5 m 16.

~ ~ ,

unpolarized lig~~~~~~~~~ (!t:' 30°

--......~

--- ....

, ~~

Polaroid: ~ ~ ~ ~

A horizontal beam of unpolarized light is incident normally upon a Polaroid, whose axis of polarization makes an angle of 30° with the vertical as shown. The transmitted beam has an intensity I. What would be the intensity of the transmitted beam if the Polaroid is rotated clockwisely through 15°?

A. I
B. I
.J2
c. I
-
2
D. I
4 17. The figure shows an object placed in front of a convex lens of focal length 20 em, A sharp image is captured on a screen placed at a certain position.

lens

object

Which statements are correct?

(1) If the object and the image are equal in size, their separation is 80 cm.

(2) If the central part of the lens is covered with a small coin, an image of the same size could still be captured.

(3) If the object is moved slightly farther away from the lens, a smaller sharp image can be captured on a screen when its position is suitably adjusted.

A. B. C. D.

(1) and (2) only (1) and (3) only (2) and (3) only (1), (2) and (3)

2010-AL-PHY 2-7

106

18. A microscope and a telescope are each made up of two converging lenses. In which of the following ways are they similar when both are in normal adjustment?

A. The separation of the two lenses is equal to the sum of their focal lengths in each case.

B. The objective has a longer focal length than the eyepiece in each case.

C. The intermediate image is magnified and inverted in each case.

D. The final image is inverted and virtual in each case.

19. Sound waves are emitted from a source at rest on the ground. When an observer is moving towards the source, which of the following quantities would increase?

(1) The wavelength of the sound waves.

(2) The speed of the sound waves relative to the ground.

(3) The frequency of the sound waves detected by the observer.

A. (I) only

B. (3) only

C. (I) and (2) only

D. (2) and (3) only

20.

A net positive charge is given to a conductor of non-uniformly curved surface. When the charge distribution on the conductor reaches electrostatic equilibrium, which statement is INCORRECT?

A. The electric field within the conductor is zero.

B. The charge density is higher at sharp points on the conductor's surface.

C. The electric potential is higher at sharp points on the conductor's surface.

D. The electric field at the conductor's surface is perpendicular to the surface.

21.

-30V

-20 V

-lOY OV

In the above figure, the four dotted lines represent equipotential surfaces in a uniform electric field. The curve is part of the trajectory of a charged particle moving inside the field. If the particle is only subject to the electric force due to the field, which of the following must be correct?

(1) The particle carries positive charge.

(2) The particle experiences the same force at X and at Y. (3) The particle has smaller kinetic energy at X than at Y.

A. B. C. D.

(1) only (3) only

(I) and (2) only (2) and (3) only

2010-AL-PHY 2-8

107

22. In the arrangement shown, points X and Yare at distances d and 2d from a fixed point charge +Q respectively. A small charge +q placed at X would have electrical potential energy U and experience electric force F.

+Q

X

Y

d --------~~-------

d

If a small charge +2q is placed at Y instead, what would be its electrical potential energy and the electric force it experienced? (Take electrical potential energy to be zero at infinity.)

electrical potential energy electric force
A. 2U '!_F
2
B. 2U F
C. U '!_F
2
D. U F 23.

b

In the above network, the resistance of each resistor is 60 O. Find the equivalent resistance of the network across a and b.

A. B. C. D.

120 150 200 240

24.

,--------f. - - -I.,___----,

Q

p'----l

In the above circuit, the battery has constant e.m.f. and negligible internal resistance. Which statement is INCORRECT when switch S is closed?

A. B. C. D.

The potential difference between P and Q remains unchanged. The electric potential at P increases.

The electric potential at Q increases.

The current flowing through resistor R2 increases.

20 I O-AL-PHY 2-9

108

25.

IIA 1.2 1.0 0.8 0.6 0.4 0.2

y
V P
/V
/ - I-- X
V v-
J """
If/ X

y

o

2

4

6

VN

Two bulbs X and Yare marked as '6 V, 3 W' and '6 V, 6 W' respectively and their I-V characteristics are shown in the above graph. When X and Y are connected in series to a d.c. supply, one of the bulbs operates at its normal rating while the other is lit dimly. Which bulb is dimly lit and what is the power consumed by it ?

bulb dimly lit power consumed
A. X 1.0 W
B. X 1.5 W
C. Y LOW
D. Y 1.5 W
26.
x x x x x x x x x x x
X
x x x x x
x >p QX x

x x x x x
x x x x x
x .j? x x
x x r x x x x x x x x In the figure, PQ and RS are two parallel metal rails with separation L. A metal rod XY resting on the rails moves with velocity v perpendicular to its length across a uniform magnetic field of flux density B pointing into the paper as shown. If the rod makes an angle 0 with the rails, what is the potential difference across Q and S? (Assume that the rod is always in contact with the rails.)

A. BLv
cosO
B. BLv cos 0
C. BLv
sinO
D. BLv sin 0 2010-AL-PHY 2-10

109

27.

6V

Two capacitors C, and C2, each of capacitance 2 uf', are connected to a cell of e.m.f. 6 V and negligible internal resistance as shown. How would the charges stored in C, and C2 change after the switch S is closed and a steady state is reached?

capacitor C1 capacitor C2
A. decreased by 4 f.lC increased by 4 f.lC
B. decreased by 4 f.lC decreased by 4 f.lC
C. decreased by 8 f.lC increased by 8 f.lC
D. decreased by 8 j.!C decreased by 8 f.lC 28. Charged particles are deflected by a magnetic field when they travel at right angles to the field. This deflection can be increased by increasing

(1) the mass of the particles. (2) the charge of the particles.

(3) the flux density of the magnetic field.

A. B. C. D.

(1) only (3) only

(1) and (2) only (2) and (3) only

29.

x

y

30

Two cells X and Y of the same e.m.f. are connected in parallel to a load of resistance 3 O. X has internal resistance 2 0 while Y has internal resistance 3 O. Which cell

(I) delivers greater power to the load, (2) dissipates greater power in itself?

(1) (2)
A. X X
B. X Y
C. Y X
D. Y Y
201O-AL-PHY 2-11 110 30.

Q

, , , , ,

, .1 ••...

,

x: x x j

: x x: x x:

R l x x:x xi : x x: x x ]

...................... i

,

'a'

In the figure, PQRS is a small rectangular metal frame suspended from a fixed point 0 by a thin rigid rod. The frame is released from the position shown and it swings about 0 in a plane normal to a uniform magnetic field pointing into the paper within the dotted rectangle. Neglect air resistance and friction. Which of the following is/are correct?

(I) A current is induced in the frame in the direction PQRSP when it is entering the field. (2) The current induced in the frame is at a maximum when it passes 00'.

(3) The direction of the magnetic force experienced by the frame is opposite to its motion when it passes 00'.

A. B. C. D.

(1) only

(1) and (2) only (2) and (3) only (1), (2) and (3)

3l.

c

a

R

b

A parallel-plate capacitor C and a resistor R are connected in series to a cell of constant e.m.f and negligible internal resistance as shown. When the plate separation of the capacitor is increased, which of the following is/are correct?

(1) A current will flow through R from a to b momentarily. (2) The energy stored in the capacitor will increase.

(3) Chemical energy is converted to electrical energy by the cell.

A. B. C. D.

(1) only

(1) and (2) only (2) and (3) only (1), (2) and (3)

2010-AL-PHY 2-12

111

32.

.ri .

A rectangular pulse is fed to the input terminals PQ of each of the circuits X and Y shown below.

circuit X

circuit Y

L

The possible resulting pulses across the output terminals ST are indicated as (1), (2) and (3).

(I)

(3)

_I.L.

(2)

Which of the following is a correct match of the circuits and the output pulses?

circuit X

circuit Y

A. B. C. D.

(1) (1) (2) (3)

(2) (3) (1) (2)

33.

100 n

11 H

In the circuit shown, the frequency of the a.c. supply is gradually increased from 100 Hz to 1000 Hz while its r.m.s. output voltage remains unchanged. As the frequency is increased, the r.m.s. current flowing in the circuit will

A. B. C. D.

increase at first and then decrease. decrease at first and then increase. increase continuously.

decrease continuously.

2010-AL-PHY 2-13

112

34.

L

The inductor in the LR circuit shown has no resistance. When the switch S is closed, the cell of e.m.f. e and with negligible internal resistance drives a current 1 in a clockwise direction, which is taken to be positive. Which equation is correct?

c.

dl e+L-+IR=O dt

e+L dJ -IR =0 dt

e-L dJ +IR =0 dt

e-L dJ -IR =0 dt

A.

B.

D.

35.

Monochromatic light is incident on a photo-emissive cell connected to a variable d.c. supply as shown. The galvanometer shows no deflection. Which of the following can be a possible reason?

(1) The temperature of the photo-emissive cell is too low. (2) The wavelength of the incident light is too long.

(3) The d.c. voltage applied has been reduced to zero.

A. (1) only

B. (2) only

C. (1) and (3) only

D. (2) and (3) only

36. In photoelectric emission experiments, when monochromatic light of wavelength A, is incident on metals X and Y, the maximum kinetic energy of the photoelectrons emitted are 1.0 eV and 0.5 eV respectively. If

the incident light is replaced by that of wavelength ~,the maximum kinetic energy of the photoelectrons 2

emitted from metal X becomes 3.0 eY. What is the maximum kinetic energy of the photoelectrons emitted from metal Y?

A. B. C. D.

1.0eV 1.5 eV 2.0 eV 2.5 eV

2010-AL-PHY 2-14

113

37.

The diagram shows the energy levels available to the outer electron in an atom of a certain element drawn approximately to scale. II.I. 11.2, 11.3 and 11.4 denote the respective wavelengths of the emitted photons corresponding to the electron transitions indicated. Which line spectrum shown on a linear scale of wavelength is correct?

A.

increasing wavelength 11.,

II I

B.

I II

c.

II I

D.

I II

38.

energy

r




4 11.3 11.2 11., ground state

In the above figure, a molecule P is fixed at the origin while another molecule Q moves along the x-axis. The intermolecular potential energy U between the two molecules varies with the separation x between them as shown. If Q is released from rest at infinity and approaches P, which of the following deductions is/are correct?

(1) Q has the greatest acceleration at x = X2' (2) Q has kinetic energy equal to U; at x = X2' (3) Q can reach a position with x < x,.

A. B. C. D.

(2) only (3) only

(1) and (2) only (1) and (3) only

20 1 O-AL-PHY 2-15

114

39.

~r-"""-=-=-,-=-- .:..:- -:..:;,-.::..;- -:..::-:.=..- ::...::- -:..:,-..:_;- -:.:-..:_- -:':-:":6:":- -::-':':~~:':~c":_~:":- -:..:,-::...::- -:..::-~- -::..:;-=- =- ==-~I

A uniform steel wire, 2.0 m long and 1.0 mm in diameter, with its ends attached to two rigid supports is held just taut horizontally. When a block is hung from the mid-point of the wire, it produces a sag of 2.0 em at that point. Find the tension in the wire. Neglect the weight of the wire itself.

Given: Young modulus of steel = 2.0 x 1011 Pa

A. 15.7 N

B. 31.4 N

C. 62.8 N

D. 126 N

40. An a-particle (~He) of initial kinetic energy 7.7 MeV approaches a gold nucleus e~~Au) from far away.

The a-particle is deflected as shown. Estimate the kinetic energy of the a-particle at point P where it is 6.5 x 10-14 m from the gold nucleus which is assumed to be stationary throughout.

Given: charge of an electron = 1.60 x 10-19 C permittivity offree space = 8.85 x 10-12 F m "

a-particle ... --=;~ -

\ \

• gold nucleus

A. B. C. D.

3.5 MeV 4.2 MeV 7.3 MeV 11.2 MeV

41.

The figure shows a horizontal Pitot tube in which a non-viscous, incompressible liquid is flowing. If the speed of liquid flow at N is 1.2 m s -I, find the difference in height of the liquid levels in the vertical tubes atMand atN.

Given: the density of the liquid = 1000 kg m-3

the cross-sectional area of the Pitot tube at M = 40 cm2 the cross-sectional area of the Pitot tube at N = 20 cnr'

A. 1.8cm
B. 3.6 em
C. 5.4cm
D. 7.2cm 2010-AL-PHY 2-16

115

42. A cancer research project employs a certain radioactive source having a half-life of 5.3 years. The source has been prepared for some time. Now the research treatment requires 10 minutes of irradiation on rats with cancer using the source. If two years later the same source is to be used for this kind of treatment, estimate the irradiation time required in order to have the same radiation dosage.

A. 13 minutes

B. 15 minutes

c. 18 minutes

D. It cannot be estimated as the time when the source was freshly prepared is not known.

43. It is known that the background count rate is 20 min-I. A GM tube is placed 8 em from a small 'Y-source, the count rate registered is 240 min-I. If the GM tube is placed 20 em from the source, what would be the approximate count rate in min-I?

A. B. c. D.

35 55 88 108

44.

pressure plPa

~y

o L..- .... temperature TIK

A fixed mass of an ideal gas changes its state from X to Y shown in the above p- T diagram. Which conclusion can be drawn?

A. The gas has to do work on the surroundings and absorb heat from the surroundings.

B. The gas has to do work on the surroundings and liberate heat to the surroundings.

c. Work has to be done on the gas while it absorbs heat from the surroundings.

D. No work has to be done by the gas while it absorbs heat from the surroundings.

45. Slotted masses are attached to the lower end of a spring clamped vertically to a stand. The system is set to perform simple harmonic motion. The period T is measured for different slotted masses m. A graph of T2 against m is then plotted.

slotted mass

o L..- + m/kg

The straight-line graph does not pass through the origin as shown. The possible reason is that

A. air resistance is present.

B. the number of oscillations is miscounted, say, time for 19 oscillations is taken as that for 20 oscillations.

C. the mass of the spring is not negligible.

D. the change in the centre of gravity of different slotted masses is not taken into account.

END OF SECTION A

20 1 O-AL-PHY 2-17

116

Section B

Answer any THREE questions from this section. Where necessary, take the acceleration due to gravity to be 10m S-2 and take the speed oflight in air to be 3 x 108 m S-I.

1. (a) Figure 1.1 shows a light spring of force constant k firmly fixed at its upper end. When a mass m is attached to its lower end, the extension of the spring is e at static equilibrium.

Figure 1.1

The mass is displaced downwards from the equilibrium position and then released. Subsequently the mass-spring system oscillates freely such that the magnitude of the displacement y of the mass from the equilibrium position is always smaller than e. (Neglect air resistance.)

(i) Show that the motion is simple harmonic and give its period in terms of m and k. Describe the phase relationship between the displacement, velocity and acceleration of the mass.

(ii) Take the gravitational potential energy to be zero at the equilibrium position. Show that the sum of elastic and gravitational potential energy of the system is given by U = _!. ky2 + _!. ke". Sketch

2 2

on the same graph the variations of the potential energy U, the kinetic energy K and the total mechanical energy E of the system with displacement y. Label the minimum value of U on your graph.

(iii) Describe and explain the effect(s), if any, on the period in each of the following cases:

(I) The system is set up on a planet where the acceleration due to gravity is only half of that on the Earth's surface.

(II) The spring is cut shorter.

(b) The mass-spring system is being forced into sinusoidal oscillation at steady state by a driving force Fa cosmt as shown in Figure 1.2.

(i)

Sketch the variation of the amplitude of oscillation A of the system with the driving frequency co for light damping. On the same graph, sketch and label the curves respectively for (I) heavier damping; and (II) a spring of larger force constant is used.

(ii) Both the free oscillation in (a) and the forced oscillation here are sinusoidal in time. Discuss the factor(s) that the amplitude and frequency of oscillation depend on in each case.

20 1 O-AL-PHY 2-18

117

(10 marks)

Figure 1.2

(6 marks)

2.

(a)

With the aid of diagrams, describe respectively the situations in which the reflection of a transverse pulse on a slinky spring undergoes (i) a phase change of n; and (ii) no phase change. (2 marks)

(b) Figure 2.1(a) shows a soap film contained in a vertical circular hoop. In a dark room, when the film is illuminated from the left with monochromatic light of wavelength A. in air, interference pattern can be seen from the same side. The refractive index of soap solution is n and near normal incidence is assumed.

monochromatic light

soap film contained in a vertical circular hoop

monochromatic ~ _ : so.ap film of

light ) _~¥thlckness t

I

Figure 2.1(a)

Figure 2.1(b)

(i) Figure 2.1 (b) shows the cross-section of a small part of the soap film of thickness t. Derive, with explanation(s), the conditions for the monochromatic light to have destructive interference at that position when observed from the left.

(ii) Because of draining of soap solution towards the bottom, the soap film will become very thin near the top and the thickness increases gradually downwards as shown in Figure 2.2(a). The pattern observed at a certain moment is shown in Figure 2.2(b).

dark bands

Figure 2.2(a)

Figure 2.2(b)

Explain why there is always a dark area at the top of the film and the separation between adjacent dark bands decreases towards the bottom. Describe the change(s) to the pattern at a later time

before the soap film breaks.

(7 marks)

(c) (i) Account for the presence of dark lines in the solar spectrum. Hence explain how we are able to

deduce the composition of the Sun's atmosphere.

(ii) Describe and explain a situation in which a spectrum observed by a stationary observer on the Earth from a source shows red shift. Hence, or otherwise, describe a method to measure the angular speed of rotation of the Sun. (Assume that the diameter of the Sun is known.)

(7 marks)

2010-AL-PHY 2-19

118

3.

(a)

In Figure 3.1, a lamp P of rating' 6 V, 50 rnA' is connected in series with a 6 V cell and a capacitor C. When switch K is closed, the lamp P glows at first. Then it becomes dimmer and goes off within about 10 s.

Figure 3.1

K

.....,~C

P

(i) By considering the voltage across the capacitor, explain qualitatively the observation described.

Estimate the order of magnitude of the capacitance of C.

(ii) If the lamp P is replaced by another one of '6 V, 100 rnA', how would the observation described be affected? Explain.

(iii) The lamp is now replaced by a resistor R of 120 n and several voltage readings VR across it at different time t during charging are recorded. Describe how you could use the data to obtain the capacitance of C using a graphical method and state the advantage of replacing the lamp by a resistor in the measurement of capacitance.

(7 marks)

(b) Figure 3.2 shows a circuit consisting of a d.c. motor M, the lamp P, an ammeter A (centre-zero) and a 6 V cell. A heavy wheel is fitted to the shaft of the motor.

6V X

....----ll-I -----<0

Figure 3.2

s

P

(i) Sketch graphs to show the respective variation of the ammeter reading I and the angular speed of the motor OJ with time right after switch S is switched to X. Explain the shape of the graphs.

(ii) When the motor is running steadily, S is quickly switched from X to Y. The heavy wheel fitted to the motor continues to rotate for some time before coming to rest. Describe and explain the observations of the lamp P and the ammeter A.

(iii) State and explain the difference, if any, in the rotation of the heavy wheel compared to (ii) if this time S is quickly switched fromXto the open position shown in Figure 3.2.

(9 marks)

2010-AL-PHY 2-20

119

4. (a)

Radioactive decay can be simulated by throwing a certain number of dice, say 400.

(i) Describe how the simulation experiment is done. The number of undecayed nuclei in radioactive decay can be expressed as N = No «". In your description, state clearly which measurements are representing N, t and dN/dt respectively and explain how the 'half-life' of dice throwing can be obtained graphically.

(ii) Would the data of this simulation experiment deviate more from the exponential law of decay if the initial number of dice is smaller, say 40 instead of 400? Explain.

(iii) Describe in words TWO properties or assumptions common to radioactive decay and dice throwing that lead to exponential law of decay.

(8 marks)

(b) (i) Some energy production processes are chain reactions. Once triggered, the process continues on

its own. Explain how the chain reaction in nuclear fission is triggered and controlled.

(ii) The schematic diagram below shows a type of nuclear power plant employing pressurized water reactor in which the reactor core is circulated by high-pressure water.

heat

pressurized water circulating

r--- ~ To the grid

~ through a step-up transformer

Figure 4.1

(1) State TWO major functions of high-pressure water in this type of reactor.

(II) Describe the energy change in the fission of uranium. Explain how the energy released in fission can be extracted from the reactor core for driving the turbine.

(III) State TWO safety measures for protecting people working in a nuclear power plant.

(8 marks)

END OF PAPER

20JO-AL-PHY 2-21

120

AI.

u2 2

a=-=OJ r

r

A2.

A3.

Gmlm2 F =--':-=-

r2

U=_GMm r

r3 / T2 = constant

A4.

A5.

Bl.

u=~

B2.

u=f!

B3.

B4.

d= AD

a

B5.

d sin B = n,A_,

B6.

B7.

BS.

I I I -+-=-

U v f

E= Q

4m;or2

V=~

4Jr&or

E=!::_

d

C=Q=&oA V d

Cl.

C2.

C3.

C4.

C5.

Q_ Q -IIRC - oe

Useful Formulae in Advanced Level Physics

centripetal acceleration

simple harmonic motion

Newton's law of gravitation

gravitational potential energy

Kepler's third law

velocity of transverse wave motion in a stretched string

velocity of longitudinal wave motion in a solid

refractive index and polarizing angle

fringe width in double-slit interference

diffraction grating equation

Doppler frequency

definition of the decibel

equation for a single lens

electric field strength due to a point charge

electric potential due to a point charge

electric field between parallel plates (numerically)

capacitance of a parallel-plate capacitor

decay of charge with time when a capacitor discharges

C6.

Q = Qo (1- e -Ii RC) rise of charge with time when charging D8. a capacitor

C7.

E=.!.CV2 2

I =nAuQ

R= pI

A

CS.

C9.

CIO. F=BQusinB

CII. F = Bll sin B

V= B1 C12.

nQt

energy stored in a capacitor

general current flow equation

resistance and resistivity

force on a moving charge in a magnetic field

force on a current-carrying conductor in a magnetic field

Hall voltage

09.

010.

C13.

B= flo1 27rr

B = floN1 I flo1112

F = --'---'--"-

27rr

C14.

C15.

C16.

T= BAN1sinifJ

C17.

E = BANOJsinOJt

CIS.

Vs ». -"='--

Vp Np

C19.

dcJ> dI

&=--=-L-

dt dt

E='!'LJ2

2

C20.

C21.

XL = OJL

magnetic field due to a long straight wire

magnetic field inside a long solenoid

force per unit length between long parallel straight current-carrying conductors

torque on a rectangular current-carrying coil in a uniform magnetic field

simple generator e.m.f.

ratio of secondary voltage to primary voltage in a transformer

induced e.m.f.

energy stored in an inductor

reactance of an inductor

C22.

1

Xc = -- reactance of a capacitor

me

C23.

P = IV cos B power in an a.c. circuit

Dl.

p V = nRT = NkT equation of state for an ideal gas

02.

03.

D4.

D5.

D6.

07.

1

P+-pu2 +pgh 2

= constant

A.U = Q+W

E = - 13.6 eV

n n2

In2 OIl. t i

2 k

.!.mu2 = hv-cJ> 2 m

012.

013.

121

kinetic theory equation

molecular kinetic energy

macroscopic definition modulus

of

Young

energy stored in stretching

relationship between force and potential energy

Bernoulli's equation

first law of thermodynamics

energy level equation for hydrogen atom

law of radioactive decay

half-life and decay constant

Einstein's photoelectric equation

mass-energy relationship

Marking Scheme

The answers provided in the marking scheme are for reference only. They are not the only possible answers. Alternative answers are acceptable so long as they are well reasoned.

The examination emphasises the testing of understanding, the practical application of knowledge and the use of processing skills. Candidates are advised to study this document in conjunction with the examiner's comments on candidates' performance in this booklet.

For essay-type questions, candidates are expected to demonstrate an understanding of the question, an ability to deploy relevant knowledge of the subject in response to the questions, and to present their answers logically and coherently.

Advanced Level Paper 1 Section A

1. (a)

1 2 mgr(l-cos B) = - mv

2

v2 = 2gr(l-cos B) or v = ~2 gr(1 - cos 0)

1M

lA ~

(b)

reaction R at Y (correct direction)

weight mg at Y (correct direction or its components)

lA lA

(c)

2 Centripetal force = mgcos 0 - R = mv r

R = mg cosO - 2mg(1 - cosB) [From (a)] = (3 cosO - 2) mg

A=3,B=2

1M

1M

R decreases as 0 increases since cos 0 decreases with O.

lA lA

1M

(d) (i) R = (3 cosOo - 2) mg = 0 when the bead leaves the hemisphere

2 cos (), = - o 3

00 = 48.2°

lA

v

The bead leaves the hemisphere with a speed tangential to its surface and moves in a parabolic path.

lA lA

4

(ii) By conservation of energy,

X~Z: ..!_mv2 =mgr 2

v2 = 2(10 m s-2)(0.1 m)

v = 1.41 m s -1 when hitting the ground or The bead leaves the hemisphere with speed u.; X ~ Y: u; = 2gr (I-cos (0)

= 2 (10)(0.1)(1-~)

3

1M

lA

2 3

171

Vertical motion: initial velocity u., sin 00, s =: r cos 00 Y~Z: v~=(uosinOo)2+2g(rcosOo)

=: 3_ sirr' 48.20 + 2(10)(0.1) cos 48.20 3

v y = 1.31 m s -1 when hitting the ground

.'. v2 = (uo cos 48.20)2 + v~

2

v2 = - cos ' 48.20 + (1.31)2 3

v =La l m s'

Or By conservation of energy,

1 2 1 2

-muo + mg(rcos (0) =: -mv

2 2

v2 = u; + 2grcos 00

v2 = 3_ + 2(10)(0.1)( 3_)

3 3

v=I.41ms-1

2. (a) (i)

Order of maxima n 1 2 3 4
Angular position of maxima () / 0 6.28 12.73 19.44 26.20
sin 0 0.109 0.220 0.333 0.442 o

2

3

4

Correct axis and scale

Points correctly plotted with best fit straight line

A

Slope of graph = - = 0.11

d

10-3

,1,=0.11 x(-)m

160

= 6.88 x 10-7 m or 688 nm

172

1M

lA

lA lA

order n

lA lA

1M

lA

Marks
2. (a) (ii) Correct set-up employing a screen.
diffracted images
IA
~~~ screen
laser _.-J' ~ grating
Safety precaution: IA
Do not view the laser light directly with eyes.
Q.! Do not point laser light towards other people.
Set up the apparatus before switching on the laser.
Precautions for getting accurate results: IA
Grating/screen should be perpendicular to light path.
Q.! Grating-screen separation should be large enough
(> 1 m) for precision measurement.
Measuring the position of maxima on both sides of the screen.
(Accept other reasonable answers) 1
(b) (i) No. of photons emitted per second:
8.0 x 1O-3C S-I
1.60xl0-19C 1M
=5.0xl016 lA ~
(ii) E= he
A- IM
1.85 eV = (1.85 x 1.60 x 1O-19)J = h(3xl08m S-I)
6.88xl0-7 m
h = 6.79 X 10-34 J s lA l
3. (a) (i) When stationary waves are formed in the Kundt's tube, lycopodium powder at the antinodes IA
is agitated whereas the powder at the nodes accumulates since the air molecules there are not lA
vibrating. ~
(ii) air pressure
normal 2A
pressure
distance along ~

the tube
(b) (i) When a stationary wave is set up, successive heaps are at a distance Y2Jl, apart.
Hence, ..1. = 2d where d = separation between successive heaps
Speed of sound in air:
e= f..1.=2fd 1M
0.35 m
= 2 (2000 Hz) (5 -1)
= 350 m S-I lA ~ 173

Marks
3. (b) (ii) The peak of the heaps are not sharp, therefore the distance between successive heaps
measured is not so accurate. IA
max. % error in c = max. % error in f + max. % error in d
0.02 m 1M
= 2% + ( )xIOO %
0.35 m
= 7.7%"" 8% IA 2.
(iii) The heap pattern is formed as a result of stationary waves of sound, which depends on the IA
frequency but not the loudness of the sound.
Therefore, there is no change in the heap positions. lA ~
(c) Intensity of background noise = In
60 dB = 10 log ( ~ J, where 10 is the intensity of threshold of hearing
In = 106 10 = Is = intensity of sound generated by the loudspeaker 1M
Combined intensity = In + Is = 2 X 106 10
[2XI06I)
Sound intensity level = 10 log 10 0
= 63 dB lA
or increase in sound intensity level = 10 log 21 = 3 dB
I
:. resulting sound intensity level = (60 + 3) dB = 63 dB ~
4. (a) (i) 222 Rn 7 218 P + 4 H lA
86 8402e
222.0176 u - (218.0090 u + 4.0026 u) = 0.006 u 1M
Energy released I1E = 931 MeV x 0.006 u
= 5.586 MeV lA 2.
(ii) The mass ratio of the products is mpo : m.; = 218: 4.
By conservation of momentum, the speed ratio is therefore VPo : Va = 4: 218. 1M
The kinetic energy ratio of the products is KEpo: KEa = 218(4)2: 4(218)2 = 4: 218. 1M
Thus nearly all (more than 98% of) the energy released goes to the alpha particle.
218 1 2
:. Mx- = -mv
222 2 1M
(5.586xl06 x 1.60xI0-19) x 218 = ..!.. (4.0026 x 1.66 x 10-27) v2
222 2
V = 1.63 X 107 m S-I IA .4
(b) (i) mv2
qvB=-- 1M
r
mv
r=-
qB
1.63x107ms-1
(4.82 x 107 C)(0.5 T)
= 0.676 m IA ~
(ii) No. Since the magnetic force (qvB) acting on the particle is always perpendicular to its IA
velocity, no work is done on the particle by the field. IA ~
(iii) When a high voltage is applied across electrodes AB, the gas/helium atoms at low pressure
would be excited by the fast bombarding electrons (or gas discharge occurs). lA
When these excited gas/helium atoms return to the ground state, several characteristic lines or lA
a characteristic spectrum would be observed, which is identical to that produced by helium in
the laboratory. IA 2.
174 Paper 1 Section B

5. (a)

GMm

mg = --2- where ra = radius of the asteroid

ra

= (14000 J kg-I){ \ )(60 kg) (from Figure 5.1)

I xlO m

= 8.4 N

(b) GMm mv2

2 GM

v =--

r

GM

At r = 2 x 105 m, v2 = -- = 7000 (from Figure 5.1) r

.·.v=83.7ms-1

T = 2nr = 2n{2 x 105 m)

v 83.7 m s-I

= 15020 s or (4.17 hours)

(c) (i) I 2 GMm

-mv =--

2 t;

vescape = ~2 GM

r;

From Figure 5.1, GM = 14000,

ra

:. ve.l'cape = ~2x14000 =167 m S-1

(ii)

v = ~3:r

,----------

3x1.38xl0-23 J K-1 x200 K

4.65 x 10-26 kg

=422 m S-I

v2 = G~ =e;rr

I

T = 2nr ~G~

T=2n:{2xl05){ b) "1/7000

T= 15020 s or (4.17 hours)

Molecular speed of gas is greater than the escape velocity of the asteroid.

Therefore, almost all gas molecules escape from the asteroid and hence no atmosphere can be retained.

175

1M

1M

IA

1M

IA

1M

lA

1M

lA

1M

1M

1A

lA

6. (a) When the electric field reaches a free electron in the rod, an electric force acts on the electron that pushes it towards one end of the rod. Opposite charges are gathered at X and Y. A voltage is set up acrossXY.

As the electric field and thus the force are changing alternately, an alternating voltage results.

(b) (i)

3.5 x 10-4 H x 5.0 x 10-11 F = 7.56 X 106 S-1

(i)

f = 2n

= 1.20 x 106 Hz (1200 kHz)

The impedance at resonance = R

= 10 n

(ii)

V

1 =_0

° Z

2.5xl0-3y 10 n

= 2.5 x 10-4 A

1 Xc=OJC

7.56xl06 S-I x5.0xl0-11 F =2.65 x 103 n

VCo = laXc

= 2.5 x 10-4 A x 2.65 x 103 n = 0.66 Y

R = 10 n

VCo = Xc

V R

Vco = Xc

V R

Vc =2.5xl0-3 X 2.65xl03

o 10

= 0.66 Y

1 Xc=OJC

(iii) When resonance occurs, the voltage across the capacitor (or the inductor) is much larger than the voltage picked up by the rod.

The selective amplification of a particular frequency enables the radio to receive only the signal of that frequency.

(c)

.S.. = C2 CocA ..

1800 - 135° 180°

C1 = 2.5 X 10-11 F (or 0.25 x 10-10 F)

176

lA

lA

1M

lA

lA

1M

1M

lA

lA

lA

1M

lA

7. (a) mg= ke

(0.5 kg)( 10 m S-2) = (50 N m-I) e e = 0.1 m

(b) (i)

(c) (i)

piston

pA = mg- ke

p(O.Ol rrr') = (0.5 kg)(10 m S-2) - (50 N m-I)(0.06 m) p = 200 Pa

Or pA = kl1x 50(0.04) P = 0.01

= 200 Pa

(ii) P1V] = P2V2

1] T2

500(0.30 x 0.01) = 200(0.24 x 0.01)

T

300 T= 937.5K

(iii)

pressure p/Pa

500

200

(ii)

'----'---'------'----''---:::c-'-!----=-'-::--~---.._. vo lume V/1 0-3 rrr'

o

W =(500+200)(3.0_2.4)xlO-3 2

= 0.21 J (W.D. by gas)

mg/sh = (0.5 kg)(10 N m-2)(0.06 m)

= 0.30 J which is greater than the W.D. by the gas

Part of the gain in gravitational p.e. (i.e. 0.30 J - 0.21 J = 0.09 J) comes from the (decrease in) elastic p.e. of the spring.

3

t,.u =-(P2V2 - PIli]) 2

= ~[500(3.0) - 200(2.4)] x 10-3 2

(iii)

= 1.53 J t,.u = Q+W Q=t,.U-W

= 1.53 - (-0.21) Q=1.74J

177

1M

1A

2A

1M

1A

1M

1A

3A

1M

1A

1A

1A

1M

1A

1M

IA

8. (a) (i)

1

uniform magnetic field B normal to the chip

(ii)

evB = eE where v = drift velocity of electrons

V

vB =E= - CD

h

G I . 1 1

enera current equation, v = -- = _-

neA neth

Sub. into equation CD : _I- B = !'_

neth h

V= BI

net

30xlO-6TxO.05 A

8.0xl019 x1.60xlO-19CxO.15xI0-3m = 0.78 mV

(b) (i)

B = J..ioi 21tr

(41t x 10-7 H m-1)(10 A)

B= .

21t (0.1 m)

= 2 x 10-5 T or 20 J.lT

(ii)

The magnetic field due to Y is directed into the paper at P while it is out of the paper at Q; whereas the Earth's magnetic field is the same (into the paper) at both P and Q. Therefore, the magnitude of the Hall voltage is larger at P where the magnetic field due to Yand the Earth's field are in the same direction.

&!: B (at P) = (30 + 20) J.lT

B (at Q) = (30 - 20) J.lT]

Paper 2 Section B

I. (a) (i) Static equilibrium: mg = ke

Restoring force = k (e + y) - mg = ky (upward) .', my = -ky (s.h.m.)

T = 2;r; = 21t 1m

(0 VI:

Acceleration leads velocity by ~ or 90° and 2

velocity leads displacement by ~ or 90°. 2

}

(or Acceleration and displacement are opposite In phase or have a 1t or 180° phase difference.)

178

IA

1

1M

1M

1M

IA

1M

lA

IA IA

IA

I. (a)

(ii) elastic P.E. + gravitational P.E.

I 2 =-k(e+y) +(-mgy)

2

I 2 I 2

= - ke + key + - ky - key

2 2

= ..!..ke2 + ..!..ky2

2 2

Energy

potential energy U (V:! mark)

IV2 mark for minimum U l~ke2 t - - - - --

1 '

_kA2~-----

2 '

2.

kinetic energy K (V:! mark)

V:! mark for correct axes labels including -A and +A

-A 0 A Y

(iii) (I) The period T (= 2n f¥ ) depends on m and k only, therefore it is unchanged on the planet.

(II) The force constant k becomes larger, therefore m = If increases and T decreases.

(b)

light IV:! mark I damping

(i)

amplitude of forced oscillation

(ii)

Simple harmonic motion

- ca is intrinsic (m = mo), i.e. depends on k and m only [V:!]

- A depends on the total energy imparted to the system initially (or the initial displacement/velocity) [V:!+V:!]

(a)

fixed end

light thread spring

------------~

Forced oscillation

- co is equal to the driving frequency (independent of k; m and damping factor etc.) [V2+V:!]

- A depends on the amplitude of the driver, force constant, damping factor and how m close to mo [y2+V2+V:!]

A pulse towards a fixed end (or a heavy spring) is reflected with a phase change of n or 1800

A pulse towards a free end is reflected without any phase change

179

1

1

2

IV:!

2 V:!

2. (b) (i)

(c)

monochromatic light

(1) ((

(2) ::

.. soap film of

r thickness ,

I ,

, , , ,

Interference occurs between ray (1) reflected from the front and ray (2) reflected from the back of the soap film.

(Optical) path difference.between rays (I) and (2) = 2nt + A/2 as there is a phase change of 1t occurred for ray (1).

Destructive interference: 2nt + A/2 = (m + 12)A where m = 0, 1,2 ...

(ii)

A

At the top, thickness of the film smaller than (i.e. 1 ~O),

4n

therefore constructive interference does not occur.

(or As t ~ 0, destructive interference occurs.)

Consider two adjacent dark bands where the thicknesses of the wedge film are tl

and 12 respectively. :,

" " " " , ,

~

, ,

, ,

I tJ '

'" i7MbMd'

2 in»

[Note: For small angle e, 2n = B => Ax = ~ ]

Ax 2nB

As e increases gradually towards the bottom, even a smaller distance further down 12

the film will give an increase of film thickness by A/2n, thus separation between 12

dark bands Ax decreases.

As time goes on, the dark area at the top increases and moves downwards. The dark bands drift towards the bottom with decreasing separation. Width of each dark/bright band becomes narrower.

(Accept using diagrams.)

(i)

The atoms in the Sun's atmosphere absorb light of the characteristic wavelengths corresponding to the transitions within the atom, and then re-radiate with the same wavelengths in all directions.

Thus the parts of the spectrum corresponding to those wavelengths appear dark by comparison with other wavelengths not absorbed in the continuous spectrum.

The absorption/dark/Fraunhofer lines in the solar spectrum indicate the presence of certain elements in the Sun's atmosphere.

As every element has a characteristic spectrum of wavelengths, it is possible to identify the elements in the Sun's atmosphere from a study/comparison of wavelengths of the Fraunhofer (dark) lines with the spectra of known elements.

180

12+12 12 12

12 12+12

2. (c)

3. (a)

(ii) Correct situation, e.g. spectrum of galaxy shows red shift.

observer

wave of speed c

rVv

source moving at speed v

o •

As the source moves away from the Earth with a certain speed, the wavelengths ~

emitted by the source would be longer than when it is stationary, i.e. shifting towards ~

the red end in the observed spectrum.

As one edge approaches the Earth while the other recedes from the Earth, ~

the dark lines due to the same element, say, iron/sodium are displaced relative to ~

each other.

Record !J.! of the same line frequency! observed or positions of absorption lines from the east and west edges of the Sun.

The angular speed of rotation (Q) = vir) of the Sun can be estimated from the Doppler

shift fl.! (4f :=: 2v ) which depends on/proportional to v (linear speed of the Sun's

! c

either edge).

(i)

Initially the capacitor is uncharged (Q = 0, Vc :=: 0), therefore the e.m.f. (E :=: 6 V) is all across the lamp and it is brightest.

As the capacitor gradually gains charge (i.e. Vc = Q ), C

the p.d. across the lamp decreases (i.e. E - Vc) and ~

the current drops/brightness of lamp decreases.

Eventually, when the capacitor is almost fully charged, Vc = 6 V and the current as ~

well as the p.d. across the lamp become almost zero/lamp becomes off.

The lamp will be practically off after a period corresponding to, say, a few time ~

constants t = RC.

Consider one r, ( 6 V 3 ) C ~ 10 s (Accept time equals r to 10 r)

50x10- A

C ~ 83333 I1F ~ 100000 I1F (0.1 F)

(Accept order of magnitude from 10-1 (0.1 F) to 10-2 (0.01 F»

(ii) The resistance of the '6 V, 100 rnA' lamp is smaller, therefore the time constant of ~

the circuit is shorter (or half), the lamp will only be lit for a shorter time (- 5 s). ~

2

Also, the initial power ( £_ ) is doubled and therefore the lamp is brighter initially. ~+~ J.

R

(iii) Plot a graph of InVR against t, where VR is the voltage across the resistor.

Since slope = -=!._ where R = 120 n is the resistance of the resistor, the capacitance RC

of C can be found.

(Accept plotting VR against t and using fll2 = 0.69 RC to find C.)

181

O.__-__._-------.

A resistor is more suitable for measurement purposes since the lamp's resistance is not constant.

(b) (i)

1

1

mo --------------------

OL----------+

01<-----------..

At t = 0, the armature coil of the motor is stationary (m= 0) and therefore no back e.m.f. is developed,

current is maximum.

The (angular) speed of the armature coil gradually increases with time, there is back e.m.f. developed in the coil when it rotates in the magnetic field within the motor.

As 1 = C - cb (R = resistance of the circuit), current/ammeter reading decreases

R

until the motor attains its working angular speed mo and steady current I", .

(ii) The armature coil is driven to rotate by the rotating heavy wheel within the magnetic field of the motor, e.m.f. is induced in the coil and

drives a current (opposite to the original one) through the lamp P and the ammeter A. (lamp lights up/ammeter shows a deflection)

The current as well as the brightness of the lamp gradually die down as the wheel slows down while its kinetic energy becomes electrical energy.

(iii) As the circuit is open (no induced current), the

wheel's kinetic energy is mainly dissipated as work done against friction/eddy currents in the core instead of driving the lamp.

('.: no induced current, :. no magnetic torque opposing the motion)

The heavy wheel will therefore rotate a longer time before it comes to rest.

182

4. (a)

(i)

Throw No = 400 dice into a confined area, say a box, and remove all those with a certain number, say 1, facing up.

Record the number of dice removed and

the number of dice remaining.

Repeat the procedure several times.

t = the number of throws

-dN/dt = the number of dice removed in each throw N = the number of dice remaining after each throw

Plot N against t and show how to deduce tvz from the graph

(Or Plot IdN/dtl against N (or t) and show how to deduce tl/2 from the slope k of the graph,

Or Any correct method, e.g. using log graph)

(ii) Yes, the exponential law holds only when N is large (i.e. it is a statistical result).

In the N against t graph, the chance that "half-life" tvn for N decreasing from 400 to 200 may be significantly different from the tll2 obtained for other pair of N (say, 40 to 20)

(Or IdN/dtl against N deviates from a straight line graph significantly)

Marks
Yz
Yz
Yz
Yz
Yz
Yz
Yz
Yz+Yz
4Yz
Yz+Yz
Yz 1 Y2 (iii) Both are random processes in which the dice / nuclei decay in the next time /second YZ+Y2 cannot be predicted (i.e. each having equal probability to decay).

For large N (~ Avogadro's constant NA or » 6 for dice), the rate of decay (or the number of a particular number facing up) at any time is proportional to the number of undecayed nuclei (or the total number of dice) present at that time.

(Or In both processes N or no. of dice becomes half after a constant t1l2)

(b)

(i)

Fission is triggered when a U-235 nucleus captures a slow neutron and split into two medium sized nuclei and several neutrons.

Control rods made from neutron-absorbing materials are inserted into the reactor core to absorb neutrons such that one fission neutron induces subsequent fission and maintains a steady chain reaction.

(ii) (I)

As moderator to slow down the fast fission neutrons so as to provide slow neutrons for further fission/chain reaction.

As coolant to take away the large amount of internal energy of the fission products.

Or As heat carrier to take away the energy from the reactor core to the heat exchanger.

(II) Fission of uranium (U-235) results in a mass defect. According to Einstein's relation (E = me2), energy is liberated and becomes kinetic energy of the fission fragments,

which heats up the pressurized water.

This energy is then extracted by the heat exchanger to produce steam (in the secondary loop), which expands and does work on the turbine.

(III) wear film badges to monitor individual dose continuously

- monitor the radiation level of the work place to ensure it is comparable to that in other industry such as cabin crew in flights

- remote or master-and-slave (robotic) equipment for operation (such as changing fuel rods) inside the core

thick steel/concrete shielding enclosing the core (Accept other reasonable answers.)

183

2
Yz+Yz
Yz
Y2
Y2 2Yz
Yz+Y2
Yz+Y2
2
Y2
Y2
Yz
Yz
Yz 2Y2
ANY
TWO
@Yz
1 Advanced Supplementary Level Paper 1

l.

(a)

1 2 mgr (l - cosO) = - mv

2

v2 = 2gr(l-cosO) or v = ~2gr(l- cose')

1M

lA

(b)

one mark for reaction R at Y (correct direction) lA
one mark for weight mg at Y (correct direction or its components) lA f.
(c) mv2
Centripetal force = mgcos(} - R = -- 1M
r
R = mg cos(} - 2mg(1 - cosO) [From (a)] 1M
= (3 cos() - 2) mg
A=3,B=2 IA
R decreases as () increases since cos () decreases with (). IA 1
(d) (i) R = (3 cos(}o - 2) mg = 0 when the bead leaves the hemisphere 1M
2
cos (l = -
o 3
(}o = 48.2° IA vy

The bead leaves the hemisphere with a velocity tangential to its surface and moves in a parabolic path.

IA IA

(ii) By conservation of energy, 1

X~Z: -mv2 =mgr 2

v2 = 2(10 m S-2)(0.1 m)

v = 1.41 m S-I when hitting the ground

1M

IA

or The bead leaves the hemisphere with speed Uo. X~ Y: u; =2gr(l-cosBo)

= 2 (10)(0.1)(1-3.)

3

2 2

Uo =-

3

184

Vertical motion: initial velocity u., sin 00, s = r cos 00 Y~Z: v; =(uosinOo)2 +2g(rcosOo)

= ~ sirr' 48.2° + 2(10)(0.1) cos 48.2° 3

Vy = 1.31 m S-I when hitting the ground

:. v2 = (uo cos 48.2°)2 + v;

v2 = ~ cos ' 48.2° + (1.31)2

3

v=I.4lms-1

Or By conservation of energy,

1 2 1 2

-muo + mg(rcos (0) = -mv

2 2

v2 = u; + 2grcos 00

v2 = ~ + 2(10)(0.1)(~)

3 3

v=I.41ms-1

2. (a)

Order of maxima n 1 2 3 4
Angular position of maxima 0 / ° 6.28 12.73 19.44 26.20
sin 0 0.109 0.220 0.333 0.442 sin 0

o

2

3

4

Correct axis and scale

Points correctly plotted with best fit straight line

2

Slope of graph = - = 0.11 d

10-3

2=0.11 x(--)m

160

= 6.88 x 10-7 m or 688 nm

185

1M

lA

IA lA

order n

lA lA

1M

IA

Marks
2. (b) Correct set-up employing a screen.
diffracted images
lA
--
~-- screen
laser ~- grating
Precautions for getting accurate results:
Grating/screen should be perpendicular to light path.
or Grating-screen separation should be large enough
(> 1 m) for precision measurement.
Measuring the position of maxima on both sides of the screen.
(Accept other reasonable answers) lA ~
3. (a) mg=ke 1M
(0.5 kg)( 10 m S-2) = (50 N rn'") e
e = 0.1 m lA z
(b) (i) pA = mg- ke

p(O.OI m2) = (0.5 kg)(10 m S-2) - (50 N m-I)(0.06 m) p = 200 Pa

[Note: pA = mg - ke (500)(0.01 )=(0.5)( 1 0)-(50)e]

e=O

(ii)

Pl~ = P2V2

1] T2

500(0.30 x 0.01) = 200(0.24 x 0.01)

T 300

T= 937.5K

(iii) pressure plPa

500

l.l. r=T+- -, .. ........... j .~.'.

200 ---- --------- ---+--~ -.. .

1 :

I '

.""_._.--. _ _.. ---- ·-····-·····t··-······-·~ , .. -

I '

'----'-_-'-----l._-'I--:::~: !---::'-;:----'-__"--' volume VII 0-3 m3

o 2.4 3.0

2A

lA

Or pA = k!u _ 50(0.04) P - om

= 200 Pa

1M

lA

1M

lA

186

3A

4. (a)

(b)

(c)

Paper 2 Section B

1. (a) (i)

B = flo! 21tr

B= (41txl0-7Hm-I)(lOA) 21t (0.1 m)

= 2 x 10-5 T or 20 IlT

d<l> &=1--1 dt

=1_ NMA 1 M

= 1600(20xl0-6T)(0.5xl0-5 m2) 0.01 s

= 0.16 mV

-0.16

&lmV 0.16

o

tis

The magnetic field due to Y is varying with time and normal to the paper at P; whereas the horizontal component of the Earth's magnetic field is constant at P.

As the search coil only detects the magnetic flux change, the graph (i.e. induced e.m.f. &) is unaffected.

Static equilibrium: mg = ke

Restoring force = k (e + y) - mg = ky (upward) :. my = -ky (s.h.m.)

T = 27r = 21t 1m

ill VI;"

Acceleration leads velocity by ~ or 900 and}

velocity leads displacement by ~ or 90°.

2

(or Acceleration and displacement are opposite in phase or have a 1t or 1800 phase difference.)

187

1M

lA

1M

lA

2A

lA

lA

lYl

1. (a) (ii) elastic P.E. + gravitational P.E.

= ..!..k(e + y)2 + (-mgy)

2

1 2 1 2

=-ke +key+-ky -key

2 2

=..!..ke2 +..!..ky2

2 2

Energy

~ __ + __ .......,.total mechanical energy E (~ mark)

(b)

(i)

potential energy U (~ mark)

I~ mark for minimum U I 2

..!..kA 2

kinetic energy K (~ mark)

~~------------------------~

: ~ mark for correct axes labels including -A and +A

,

fixed end

light thread spring

-----------~

(ii)(I)

monochromatic light

(I) (( )

(2) ==

A pulse towards a fixed end (or a heavy spring) is reflected with a phase change of'n or 1800

A pulse towards a free end is reflected without any phase change

rw'p film of

-_ thickness I

, ,

, ,

, I

I I

Interference occurs between ray (1) reflected from the front and ray (2) reflected from the back of the soap film.

(Optical) path difference between rays (1) and (2) = 2nt + }J2 as there is a phase change of 1t occurred for ray (1).

Destructive interference: 2nt + }J2 = (m+~)A where m = 0, 1,2 ...

(II) At the top, thickness of the film smaller than ~ (i.e, t ~O), 4n

therefore constructive interference does not occur.

(or As t ~ 0, destructive interference occurs.)

As time goes on, the dark area at the top increases and moves downwards. The dark bands drift towards the bottom with decreasing separation. (Accept using diagrams.)

188

2\ti

\ti+\ti

\ti+\ti

\ti
\ti+\ti
\ti
\ti 3\ti
\ti
~
\ti
\ti+\ti
2\ti 2. (a) (i)

Experimental set-up

c

First set the rheostat to maximum resistance and Yz

the capacitor is shorted by a lead to ensure it is uncharged initially (while the meter Yz

reading is preferably slightly less than its f.s.d).

Remove the shorting lead, the charging process starts and the charging current starts to

decrease. At the same time, press the stop watch to start timing. Y2

During the experiment, decrease the resistance of the rheostat so as to keep the current Yz

constant (say, 10) which is monitored by the meter. Yz

Take the voltage reading across C on the eRO at different time t. Y2

Reset the rheostat to maximum value and repeat the above procedures to find the average voltage readings Vat time t so as to increase accuracy.

As Q = lot, a graph of V against t (oc Q for constant current) should be plotted which is a straight line passing through the origin.

(ii) Initially the capacitor is uncharged (Q = 0, Vc = 0), therefore the e.m.f. (8 = 6 V) is all Yz across the lamp and it is brightest.

As the capacitor gradually gains charge (i.e. Vc = Q ), Yz

C

the p.d. across the lamp decreases (i.e. 8 - Vc) and the current dropslbrightness of lamp A decreases.

Eventually, when the capacitor is almost fully charged, Vc = 6 V and the current as well as the p.d. across the lamp become almost zero/lamp A becomes off.

189

2. (b) (i)

1

or

1

mo _

0'-- ....

o ~ ~

At t = 0, the armature coil of the motor is stationary (m = 0) and therefore no back e.m.f. is developed,

current is maximum.

The (angular) speed of the armature coil gradually increases with time, there is back e.m.f. developed in the coil when it rotates in the magnetic field within the motor. As

1 = G - Gb (R = resistance of the circuit), current/ammeter reading decreases until the R

motor attains its working angular speed mo and steady current 100 .

(ii) The armature coil is driven to rotate by the rotating heavy wheel within the magnetic field of the motor, e.m.f. is induced in the coil and

drives a current (opposite to the original one) through the lamp P and the ammeter A. (lamp lights up/ammeter shows a deflection)

The current as well as the brightness of the lamp gradually die down as the wheel slows down while its kinetic energy becomes electrical energy.

(iii) As the circuit is open (no induced current), the

kinetic energy of the wheel is mainly dissipated as work done against friction/eddy currents in the core instead of driving the lamp.

(.: no induced current, :. no magnetic torque opposing the motion) It will therefore rotate a longer time before it comes to rest.

Q.3 Please refer to Q.4 of A-level Paper 2B.

190

=ti~1Jf G=EJI$
Advanced Level Paper 2 Section A
II. ~* II. ~*
Question No. Key Question No. Key
I. A(34) 26. e (30)
2. A(54) 27. B (41)
3. e (47) 28. 0(67)
4. 0(64) 29. A(l5)
5. 0(57) 30. A (76)
6. B (48) 31. A (73)
7. B (79) 32. A (52)
8. A (29) 33. 0(43)
9. B (84) 34. 0(60)
10. e (54) 35. B (60)
II. e (30) 36. 0(48)
12. B (59) 37. e (62)
13. B (35) 38. A (48)
14. e (66) 39. B (36)
15. 0(69) 40. B (42)
16. A(32) 41. 0(37)
17. D (64) 42. A(43)
18. 0(51) 43. B (43)
19. B (57) 44. A (42)
20. e (62) 45. e (56)
21. 0(28)
22. e (81)
23. e (48)
24. 0(58)
25. e (40)
IT: MIJJJtPfIi*J.'$!§JYaj}$o
Note: Figures in brackets indicate the percentages of candidates choosing the correct answers. 191

= lUi 16 ~ IJf &=IfIBB
Advanced Supplementary Level Paper 2 Section A
.M 5* aM 5*
Question No. Key Question No. Key
1. A (16) 16. C (29)
2. A(51) 17. C (31)
3. C (42) 18. B (30)
4. D (49) 19. D (67)
5. D (41) 20. A (12)
6. B (37) 21. A(62)
7. B (55) 22. B (33)
8. A(14) 23. A (31)
9. B (28) 24. *
10. C (57) 25. C (46)
11. D (54)
12. D (18)
13. C (47)
14. C (27)
15. D (49)
* :$: w: ~lHEHlIJIJ '* 0
This item was deleted. li: Ii!! f/;f pg Ii * 1$ If Jtj B:fJ' If5 0

Note: Figures in brackets indicate the percentages of candidates choosing the correct answers.

H~ rBMH~lmoi tl9iJtBJJ m~~~,~m~~&~~~~*~~~~~mM~~$¥®~M~gm'Rm W.mmm~MIDA~~om.~tt •• ,~~~~~~~~~g~;.m~~ ~ ~ 1i~ ~ M ~ fi JJU :fJ ~~ , * fjg 1~ ~ ~ fE ~ ~ ~ ~ 51 m I:B * ' ft!! § Z ' :*: ~ IQ: ~ ~ 3MWR •• ~o~Wmm~m'~ •• ~.~~~~'M~~m~mB~~o m.~M •• ~~~~~$~~~ffl'@ill~~~~MW.~'~~~.~'~ 'i'~~~~¥~15:E}i(;.ml:B~i~ifa 0

General note on item deletion

It is normal for the HKEAA to delete a small number of items from its multiple-choice question papers if they prove unsatisfactory. In practice, there are a number of reasons why this is considered necessary. By far the most common reason for deleting an item is that the item fails to discriminate between weak and able candidates - in other words, the majority of the candidates involved had to rely on guesswork in answering that question. If such an item is retained, the measurement process is rendered less effective. Where items have been deleted in the live papers, they are still included in this series of publications. They are indicated as deleted items. Such items may be discussed in the corresponding examination reports.

192

You might also like